LSAT and Law School Admissions Forum

Get expert LSAT preparation and law school admissions advice from PowerScore Test Preparation.

 Basia W
  • Posts: 108
  • Joined: Jun 19, 2014
|
#15717
Good evening,

Would you suggest using a conditional reasoning diagram for this question? And if you were to do so, how does one arrive at B? This answer choice surprised me.

Best,

Basia
 Luke Haqq
PowerScore Staff
  • PowerScore Staff
  • Posts: 722
  • Joined: Apr 26, 2012
|
#15737
Hi Basia,

That's a great question. On the one hand, it's almost always the best idea to diagram every time you see conditional reasoning. This helps one be able to identify inferences, avoid trap answers, and generally have a understanding of the stimulus. On the other, a test taker might also find that substantial time was put into diagramming out conditional statements, and they don't end up being necessary to reach the right answer.

This particular question is a tricky case since every sentence involves conditional reasoning. My advice for a question like this is to read it through once, identify the conclusion, and read the question. Since this is a flaw question, when you go back through a second time, search to identify the flaw(s).

The stimulus states that a proposal must be supported by the director to be approved. However, it does not state that the director's support guarantees it will be approved. Thus, while Agnes' proposal will be supported by the director, we do not know whether it will be approved as well, in which case we do not know that the lab must be cleaned out.
 eober
  • Posts: 107
  • Joined: Jul 24, 2014
|
#16514
Hi,

How do we diagram "only those proposals the director supports will be approved"? Only introduces the necessary condition and I thought the necessary condition is "directors support", thus:

Approved --> Director's support

Is this correct?
 BethRibet
PowerScore Staff
  • PowerScore Staff
  • Posts: 200
  • Joined: Oct 17, 2012
|
#16548
Hi Eober,

Yes, you've got this one just right!

Beth
 andbzav@gmail.com
  • Posts: 17
  • Joined: Jul 18, 2019
|
#68045
Hi,

Is it right to call the flaw in this reasoning a mistaken reversal because: approved -> director's support, and not: director's support -> approval..?

Thank you
User avatar
 KelseyWoods
PowerScore Staff
  • PowerScore Staff
  • Posts: 1079
  • Joined: Jun 26, 2013
|
#68273
Exactly right, andbzav! This stimulus present a Mistaken Reversal just as you've diagrammed. The director's support is required for approval but having the director's support is not enough to guarantee the proposal will be approved. Thus, it is possible for a proposal to be rejected even with the director's support.

Great job!

Best,
Kelsey
User avatar
 smtq123
  • Posts: 29
  • Joined: May 28, 2021
|
#92082
Hi,

Can you please explain why E is not correct?

If E is true, then the "MUST" part in the conclusion is affected.

Many thanks for your support.
 Adam Tyson
PowerScore Staff
  • PowerScore Staff
  • Posts: 5153
  • Joined: Apr 14, 2011
|
#92221
Answer E is incorrect, smtq123, because the argument does not make any such presumption. The stimulus makes clear that if Agnes' proposal is approved, the 4th floor lab must be cleaned out for her use. It doesn't matter if some other lab would be adequate; the rule has been established that cleaning out the 4th floor lab is necessary if her proposal is approved.

Think of the negation of answer E: what if there are other adequate labs? That has no impact on the conclusion, because the argument still includes the rule that approval of her proposal guarantees that the 4th floor lab must be cleaned out for her use. It's analogous in some ways to a simple contract: if you agree to sell me your car, then it makes no difference if some other car would be adequate for my needs. It's YOUR car that I bargained for, so as long as I hold up my end of the deal, it's your car that I get.

The real problem here is the mistaken reversal. A proposal being approved is sufficient to prove that the director supported it, but the director supporting it does not prove that it will be approved. Her support is necessary, but may not be sufficient as the author has treated it.

Get the most out of your LSAT Prep Plus subscription.

Analyze and track your performance with our Testing and Analytics Package.